Phương pháp UCT trong chứng minh bất đẳng thức (võ quốc bá cẩn)

33 1.5K 22
Phương pháp UCT trong chứng minh bất đẳng thức (võ quốc bá cẩn)

Đang tải... (xem toàn văn)

Tài liệu hạn chế xem trước, để xem đầy đủ mời bạn chọn Tải xuống

Thông tin tài liệu

Võ quốc bá cẩn.Các phương pháp chứng minh bất đẳng thức,Trần quốc anh.Bất đẳng thức ôn thi HSG.Luyện thi chuyên 10 .chuyên khoa học tự nhiên đại học quốc gia hà nội.Hà nội amsterdam.Chuyên lam sơn thanh hóa.Quốc học huế

Kỷ thuật hệ số không xác định (U.C.T) • Nguyễn Thúc Vũ Hoàng Học sinh chuyên Toán-Tin-THPT Chuyên Lê Quí Đôn-Niên khóa 2006-2008 Thị xã Đông Hà-Tỉnh Quảng Trị • Võ Quốc Cẩn Sinh viên K32 Khoa Dược-Đại học Y Dược Cần Thơ -Niên Khóa 2006-2011 Thành Phố Cần Thơ Có điều bí ẩn mà bạn chưa biết đến ?! Câu trả lời rất nhiều bạn cảm thấy bực bội, khó chịu tìm lời giải thích thỏa đáng cho bí ẩn Nhưng bạn quan niệm đằng sau điều hàm chứa ý nghĩa định Và ngẫu nhiên mà lí giải lại hình thành Trong giới bất đẳng thức Đôi bạn hiểu người ta lại tìm lời giải trông “kì cục” !!! Phải lần mò may rủi tìm ? Câu trả lời lại lần nữa nhắc lại: mỗi lời giải có giải thích riêng thân Việc tìm lời giải phải qua trình lập luận, thử, sai đúng Trong chuyên đề nho nhỏ chúng muốn giới thiệu đến bạn kĩ thuật không kém phần hiệu việc chứng minh số dạng bất đẳng thức Nó không giúp ta giải tất toán mà giúp ta tìm những lời giải ngắn gọn ấn tượng lớp toán Một số toán dễ đối với phương pháp lại khó đối với kỹ thuật Đây điều hiển nhiên dễ hiểu Mục lục • • • • • • • • • • Phần Bài toán mở đầu Phần Khởi đầu số toán Phần Kĩ thuật chuẩn hóa U.C.T Phần U.C.T kỹ thuật phân tách trường hợp Phần Kết hợp bất đẳng thức Vornicu Schur với U.C.T Phần Một dạng biểu diễn thú vị Phần Giải số toán mà điều kiện liên quan mật thiết đến Phần U.C.T mở rộng Phần Lời kết Phần 10 Bài tập áp dụng Phần Bài toán mở đầu Bài toán [Nguyễn Thúc Vũ Hoàng] Cho a, b, c số thực dương thỏa mãn a + b + c = Chứng minh 1 2(a + b + c ) + + + ≥5 a b2 c2 Chứng minh Ta sử dụng bất đẳng thức sau 2a 2a + ≥ − a2 3 Thật bất đẳng thức tương đương với (a − 1) (2a + 6a + 3) ≥0 3a Hiển nhiên đúng với a số thực dương Sử dụng bất đẳng thức tương tự với b c Ta có điều phải chứng minh Đẳng thức xảy a = b = c = Chắc chắn đọc lời giải cho toán “ đơn giản” bạn có phần lúng túng không hiểu lại tìm bất đẳng thức phụ cách “khó hiểu” Phải dự đoán cách “vô hướng” Hoặc có người nghĩ toán tạo từ bất đẳng thức phụ Câu trả lời hoàn toàn Tất theo qui luật Ở phần chúng phân tích kỹ thuật phân tích giúp tìm bất đẳng thức phụ mở rộng vấn đề theo chiều hướng mẻ Kỹ thuật có tên U.C.T, viết tắt chữ đầu cụm từ tiếng Anh Undefined Coefficient Technique Hay gọi Kỹ Thuật Hệ số bất định Đây kỹ thuật tảng quan trọng đường tìm kiếm lời giải cho những bất đẳng thức khó Phần Khởi đầu cùng một số toán bản Chúng ta khởi đầu kỹ thuật việc đưa cách giải thích cho việc tìm bất đẳng thức phụ cách giải thích cho toán sau chúng ta Bài toán biến vế điều kiện không ràng buộc điều khiến ta nghĩ tách theo biến để chứng minh đơn giản Nhưng rõ ràng ta không đủ Nếu ta chứng minh bất đẳng thức sau 2a (a − 1)(a + 1)(2a − 3) + ≥ ⇔ ≥0 3 a2 3a Rõ ràng không hoàn toàn đúng với a thực dương Đừng bỏ cách ta chưa sử dụng điều kiện a + b + c = Như ta không theo đường lối suy nghĩ đơn giản ban đầu nữa mà tìm hệ số để bất đẳng thức sau đúng 2a (1) + ≥ + ma + n 3 a2 Trong m n hệ số chưa xác định Tương tự với biến b c Cộng vế theo vế ta có 1 2a + 2b + 2c 5 + + + ≥ + m(a + b + c) + 3n = + 3(m + n) 2 3 a b c Như hệ số m n phải thỏa mãn điều kiện m + n = ⇔ n = −m Thế vào (1) dẫn đến 2a + ≥ + m(a − 1) (2) 3 a2 Đến ta cần xác định hệ số m để bất đẳng thức (2) đúng Chú ý toán điểm cực trị đạt a = b = c = nên ta cần xác định m cho  (a + 1)(2a − 3)  2a + ≥ + m(a − 1) ⇔ (a − 1) − m  ≥ 2 3 a 3a   (a + 1)(2a − 3) = − từ ta dự đoán m = − để tạo 3 3a thành đại lượng bình phương (a − 1) biểu thức Từ ta chứng minh bất đẳng thức phụ 2a 2a + ≥ − a2 3 Khi cho a = ta có Quá trình tìm bất đẳng thức phụ phân tích cụ thể Tuy nhiên cách để ta tìm hệ số Ta sử dụng tính chất đường tiếp tuyến điểm đồ thị hay sử dụng đạo hàm Nhưng có lẽ cách dự đoán hữu hiệu đơn giản mặt trực quan thực Tuy nhiên tất dự đoán Nó không đảm bảo sau tìm bất đẳng thức phụ toán giải Một số dạng toán đề cập phần chuyên đề Ở phần chúng ta chứng minh số bất đẳng thức đề hình thành đầu kỹ thuật qua thành thục việc phân tích Ta tiếp tục đến với toán sau Bài toán [Vasile Cirtoaje] Cho a, b, c, d số thực dương thỏa mãn a + b + c + d = Chứng minh 1 1 + + + ≥2 a +1 b +1 c +1 d +1 Chứng minh Ta xác định hệ số m để bất đẳng thức sau đúng (a − 1)(a + 1)  a +1  ≥ + m(a − 1) ⇔ − ≥ m(a − 1) ⇔ (a − 1) − − m ≥ 2 a +1 a +1  a +1  a +1 = −1 ⇒ m = −1 Ta dự đoán bất đẳng thức sau đúng thật Khi a = ta có − a +1 a(a − 1) ≥ 2−a ⇔ ≥0 a2 +1 a2 +1 Tương tự với biến lại Cộng vế theo vế ta có điều phải chứng minh Đẳng thức xảy a = b = c = d = Nhận xét Ta sử dụng kỹ thuật “Côsi ngược dấu” để tìm bất đẳng thức phụ a2 a2 a = − ≥ − = 1− 2 2a a +1 a +1 Bài toán [Algebraic Inequalities Old and New Method] Cho a, b, c số thực dương thỏa mãn a + b + c = Chứng minh 1 + + ≤1 a +b+c b +c+a c +a+b Chứng minh Ở ta cần tìm m để bất đẳng thức đúng 1 a( a − 1) = ≤ + m( a − 1) ⇔ − ≤ m(a − 1) a +b+c a −a+3 3(a − a + 3) Tương tự ta tìm dự đoán với m = − bất đẳng thức phụ đúng Thật a (a − 1) (3 − a) (a − 1) (b + c ) ≤ − ⇔ ≤ ⇔ ≤ a2 − a + 9 3(a − a + 3) 3(a − a + 3) Nhận xét Bài toán giải kĩ thuật “Phân tách Chebyshev” xem cách giải U.C.T lại đơn giản mặt ý tưởng Bài toán tổng quát giải định lí LCF “Algebraic Inequalities -Old and New method” tác giả Vasile Cirtoaje Cho a1 , a2 , , an số thực không âm thỏa mãn a1 + a2 + + an = n Chứng minh 1 + + ≤1 a1 − a1 + n a2 − a2 + n an − an + n Bài toán [Nguyễn Thúc Vũ Hoàng] Cho a, b, c, d số thực không âm thỏa a + b + c + d = Chứng minh 2(a + b + c + d ) ≥ + + ab + ac + ad + bc + bd + dc Chứng minh Theo a, b, c, d số thực dương thỏa mãn a + b2 + c + d = ⇔ ( a + b + c + d ) = 2(2 + ab + ac + ad + bc + bd + cd ) ⇔ ( a + b + c + d ) = 2(2 + ab + ac + ad + bc + bd + cd ) Bất đẳng thức cần chứng minh tương đương với 2(a + b + c + d ) ≥ + (a + b + c + d ) Ta cần xác định hệ số m để bất đẳng thức sau đúng 3a + (2a + 1) (a − 1) 2a ≥ + m(a − 1) ⇔ ≥ m(a − 1) 2 Dễ dàng dự đoán m = Ta chứng minh điều đó, 3a + 9(a − 1) 2a ≥ + ⇔ 2(a − 1) ( a + 2) ≥ 2 Điều hiển nhiên đúng Đẳng thức xảy a = b = c = d = Nhận xét Bài toán với hình thức “cồng kềnh” chứa thức Tuy nhiên nhận điểm mấu chốt toán ta dễ dàng đưa đơn lượng theo biến để giải Bài toán giải theo cách khác cách chứng minh trực tiếp với biến Nhưng dù việc giải theo biến riêng biệt dễ dàng nhiều Bài toán Cho a, b, c số thực dương thỏa mãn a + b + c = Chứng minh 1 1 4 + +  + 5(a + b + c ) ≥ 27 a b c Chứng minh Ta cần tìm hệ số m cho (a − 1)(5a + 5a − 4) + 5a ≥ + m(a − 1) ⇔ ≥ m(a − 1)(a + a + 1) a a Ta dễ dàng nhận đẳng thức xảy a = b = c = Khi cho a = ta dự đoán m = Ta chứng minh với m = bất đẳng thức phụ đúng Thật (a − 1) (−2a + a + 4) + 5a ≥ + 2a ⇔ ≥0 a a Do a ≤ 3 ⇒ −2a + a + ≥ Vậy bất đẳng thức phụ đúng Đẳng thức xảy a = b = c = Bài toán Cho a1 , a2 , , an số thực không âm thỏa mãn n ∑ 3a i =1 n ∑a i =1 i = n Chứng minh n ≤ i +5 Chứng minh Ta tìm hệ số m cho (5 − 3ai )(a i − 1) ≤ + m(a i − 1) ⇔ ≤ m(ai − 1) 3ai + 8(3ai2 + 5) Ta dự đoán với m = bất đẳng thức phụ đúng Thật vậy: 32 (5 + )(ai − 1) (a i − 1) ≤ + ⇔ ≤ 32 3ai2 + 32(3ai2 + 5) Điều hiển nhiên đúng Đẳng thức xảy biến Nhận xét Qua toán ta thấy bất đẳng thức không quan tâm đến số biến Ta hoàn toàn tổng quát với n biến mà không làm ảnh hưởng đến cách giải Đây điểm thú vị U.C.T Một cách tổng quát ta đưa cách giải cho lớp toán có dạng sau Bài toán tổng quát Cho số thực không âm a1 , a2 , , an thỏa mãn h(a1 ) + h(a2 ) + + h(an ) = Chứng minh f ( a1 ) + f (a2 ) + + f (an ) ≥ Lớp toán giải cách phân tách để chứng minh theo biến Vì biểu thức mang tính đối xứng với nên thường điểm cực trị đạt biến Ta phải xác định hệ số m cho f ( a i ) ≥ m × h( a i ) Đúng với biến thỏa mãn điều kiện đặt Với cách giải ta giải lượng lớn bất đẳng thức mà biến không ràng buộc lẫn cách “mật thiết” n k Thường số dạng điệu kiện ∑ = n Có thể khái quát tư tưởng kỹ thuật i =1 lớp toán sau: Để chứng minh toán ta xác định hệ số bất đẳng thức phụ theo biến riêng biệt cho f (ai ) ≥ m × h(ai ) ⇔ g (ai ) k p (ai ) ≥ Trong g (ai ) = (ai − x k ) với x k điểm cực trị bất đẳng thức Bài toán giải p(ai ) ≥ Trong trường hợp p(ai ) ≥ đúng miền nghiệm ta tiến hành chia trường hợp để giải toán Tuy nhiên phần ta không đề cấp đến những toán mà đề cập phần sau Sau tìm bất đẳng thức phụ Với nhiều công cụ đạo hàm, khảo sát hàm số hay đơn giản phân tích nhân tử ta giải không khó khăn Trong phép chứng minh cho bất đẳng thức phụ ta biến đổi qui việc phân tích nhân tử đa thức an x n + an −1 x n −1 + a2 x + a1 x + a0 Mà mục đích chủ đạo qui dạng tổng bình phương Việc nhân tích đa thức thành nhân tử vấn đề Đại số nên xin không nêu Qua vài ví dụ nho nhỏ hẳn phần bạn hiểu U.C.T Ở phần việc xác định hệ số trình bày cách sơ lược những toán mang tính phức tạp nhiều mà U.C.T đơn bước đệm để đến lời giải đưa ta cách chứng minh trực tiếp Phần Kĩ thuật chuẩn hóa U.C.T Bây chúng ta bước sang khoảng không gian với lớp bất đẳng thức đối xứng ba biến kĩ thuật chuẩn hóa kết hợp với U.C.T Đa thức f (a, b, c ) đối xứng định nghĩa dạng: f ( a, b, c) = f / (a / , b / , c / ) (a / , b / , c / ) hoán vị tùy ý (a, b, c) Hay nói cách khác f (a, b, c) = f (b, c, a ) = f (c, a, b) Tính đa thức đối xứng ba biến miền D có nghĩa f ( ka, kb, kc) = k n f ( a, b, c) với k , a, b, c ∈ D, n = const phụ thuộc vào hàm f ( a, b, c) Hiểu cách đơn giản đa thức tổng đơn thức đồng bậc Do số tính chất hàm ta chuẩn hóa điều kiện biến để đơn giản hóa việc chứng minh Ta chuẩn hóa đa thức đối xứng ba biến cách đặt a n + b n + c n = k , abc = p, ab + bc + ca = r , Đây kỹ thuật quan trọng giúp ta đơn giản hóa qui bất đẳng thức chứng minh theo biến Hãy đến với số bất đẳng thức đối xứng ba biến để thấy công dụng U.C.T Bài toán [Bất đẳng thức Nesbit] Cho a, b, c số thực không âm Chứng minh a b c + + ≥ b+c c+a a +b Chứng minh Không tính tổng quát chuẩn hóa a + b + c = Bài toán qui việc chứng minh a b c + + ≥ 3− a 3−b 3−c Ta cần chứng minh bất đẳng thức a 3(a − 1) ≥ + m(a − 1) ⇔ ≥ m(a − 1) 3− a 2(3 − a) Dễ dàng dự đoán m = Ta chứng minh bất đẳng thức với m đúng a 3a − 3(a − 1) ≥ ⇔ ≥0 3− a 4(3 − a ) Điều hiển nhiên đúng Sử dụng tương tự với biến lại Cộng vế theo vế ta có điều phải chứng minh Đẳng thức xảy a = b = c Nhận xét bất đẳng thức Nesbit bất đẳng thức đại số có nhiều phép chứng minh Lời giải lời giải đẹp ngắn gọn cho bất đẳng thức Bài toán [Võ Quốc Cẩn] Cho a, b, c số thực không âm Chứng minh (b + c − a ) ( a + c − b) (a + b − c) 3(a + b + c ) + + ≥ 2a + (b + c) 2b + (a + c) 2c + (b + a ) (a + b + c) Chứng minh Chuẩn hóa a + b + c = Khi bất đẳng thức cần chứng minh tương đương với 2(3 − 2a ) 2(3 − 2b) 2(3 − 2c) + + ≥ a2 + b2 + c2 a − 2a + b − 2b + c − 2c + Ta cần xác định hệ số m để bất đẳng thức sau đúng 2(3 − 2a ) ≥ a + m(a − 1) a − 2a + Ta lại có 2(3 − 2a ) (a − 1)(a + 3)(a − 4a + 6) − a = − a − 2a + a − 2a + Từ dễ dàng dự đoán với m = −6 bất đẳng thức phụ đúng Thật 2(3 − 2a ) (a − 1) (6 − a)a ≥ a − 6( a − 1) ⇔ ≥0 a − 2a + a − 2a + Điều hiển nhiên đúng a ∈ (0,3) Tương tự với biến lại Đẳng thức xảy a = b = c Bài toán [Đề thi Olympic 30-4, khối 11, lần XII – 2006] Cho a, b, c số thực dương Chứng minh a (b + c) b (c + a ) c ( a + b) + + ≤ 2 2 2 (b + c ) + a (c + a ) + b ( a + b) + c Chứng minh Không tính tổng quát, chuẩn hóa a + b + c = Ta có bất đẳng thức cần chứng minh tương đương với a (3 − a ) b(3 − b) c (3 − c ) + + ≤ 2 − 6a + 2a − 6b + 2b − 6c + 2c Tương tự ta dễ dàng tìm bất đẳng thức phụ sau: a (3 − a ) 21 + 9a (a − 1) (18a + 9) ≤ ⇔0≤ − 6a + 2a 25 25(9 − 6a + 2a ) Điều hiển nhiên đúng Đẳng thức xảy a = b = c Nhận xét Có thể thấy hai lời giải cho toán mở đầu phần đơn giản ngắn gọn Đây xem kỹ thuật thống Giúp ta giải số toán “cùng loại” quen thuộc sau Bài toán [Darij Grinberg, Old and New Inequalities] Cho a, b, c số thực dương Chứng minh a b c + + ≥ 2 (b + c ) ( c + a ) ( a + b ) 4( a + b + c) Chứng minh Không tính tổng quát, giả sử a + b + c = Bài toán cần chứng minh qui dạng sau a b c + + ≥ 2 (3 − a ) (3 − b) (3 − c) Dễ dàng dự đoán bất đẳng thức phụ sau a 2a − (a − 1) (9 − 2a) ≥ ⇔ ≥0 (3 − a ) 4(3 − a) Điều hiển nhiên đúng a ∈ [0,3) Sử dụng bất đẳng thức cho b, c cộng lại, ta có đpcm Bài toán 10 [Phạm Văn Thuận, Mathlinks forum] Cho a, b, c số thực dương Chứng minh (b + c − 3a ) (a + c − 3b) (a + b − 3c ) + + ≥ 2a + (b + c) 2b + (a + c) 2c + (b + a ) 2 Chứng minh Không tính tổng quát, chuẩn hóa a + b + c = Ta có bất đẳng thức cần chứng minh tương đương với (3 − 4a) (3 − 4b) (3 − 4c) + + ≥ 2 2 2 2a + (3 − a) 2b + (3 − b) 2c + (3 − c) Sử dụng bất đẳng thức phụ sau (3 − 4a) 8a − (a − 1) (39 − 8a) ≥ ⇔ ≥0 2a + (3 − a) 6(a − 2a + 3) Điều hiển nhiên đúng ≤ a ≤ ⇒ 39 − 8a ≥ 39 − 24 = 15 > Tương tự với biến lại ta có điều phải chứng minh Đẳng thức xảy a = b = c Bài toán 11: [USAMO 2003] Cho a, b, c số thực dương Chứng minh (b + c + 2a ) (a + c + 2b) ( a + b + 2c) + + ≤8 2a + (b + c) 2b + (a + c) 2c + (b + a ) Chứng minh Không tính tổng quát, chuẩn hóa a + b + c = Khi ta có bất đẳng thức cần chứng minh tương đương với (a + 1) (b + 1) (c + 1) + + ≤8 2a + (1 − a) 2b + (1 − b) 2c + (1 − c) Sử dụng bất đẳng thức phụ sau (a + 1) 12a + (3a − 1) (4a + 1) ≤ ⇔ ≤ 2a + (1 − a) 2a + (1 − a) Điều hiển nhiên đúng Đẳng thức xảy a = b = c Phần U.C.T kỹ thuật phân tách trường hợp Ở phần ta làm quen với số toán đưa dạng f (ai ) ≥ m × h(ai ) ⇔ g (ai ) k p (ai ) ≥ Thì có điều phải chứng minh Tuy nhiên xuất p (ai ) ≥ Trong trường hợp p (ai ) ≥ đúng với miền nghiệm việc chứng minh phải qua chiều hướng khác, xét thêm trường hợp biến miền xác định để p (ai ) ≥ Thường bước phức tạp đòi hỏi người làm phải có những đánh giá mang tinh tế nhiều Chúng ta đến với số toán tiêu biểu cho kỹ thuật Bài toán 12 Cho a, b, c số thực dương Chứng minh a2 b2 c2 + + ≥ 2 2 a + (b + c ) b + (a + c ) c + (b + a) Chứng minh Không tính tổng quát chuẩn hóa a + b + c = Qui bất đẳng thức dạng a2 b2 c2 a2 + + ≥ ⇔ ≥ ∑ 2 2 2 a + (3 − a ) b + (3 − b) c + (3 − c) 5 cyc a − a + Ta sử dụng bất đẳng thức phụ sau a2 12a − ≥ ⇔ (8a − 21)(a − 1) ≥ 2a − 6a + 25 a ≥ b ≥ c ⇒ a ≥1≥ c Không tính tổng quát giả sử Xét hai trường hợp sau 21 ⇒ 8a − 21 ≥ 8b − 21 ≥ 8c − 21 ≥ + Trường hợp c ≥ 21 + Trường hợp max{a, b, c} ≤ Khi ta có: a2 49 f (a) = = ≥ > 2a − 6a + 50 3  +  − 1÷ a  Do f ( a) đồng biến (0,3] nên điều hiển nhiên đúng Vậy toán chứng minh Đẳng thức xảy ba biến Bài toán 13 [Vasile Cirtoaje - Algebraic Inequalities – Old and New Method] Cho a, b, c, d số thực dương thỏa mãn a + b + c + d = , Chứng minh 1 1 16 + + + ≥ 3a + 3b + 3c + 3d + Chứng minh Ta cần xác định hệ số để bất đẳng thức sau đúng ≥ + m(2a − 1) 3a + Dễ dàng tìm bất đẳng thức phụ sau 3a + Tương tự với biến lại Xét hai trường hợp sau + Trường hợp ≥ min{a, b, c, d } ≥ 52 − 48a 3(2a − 1) (12a − 1) ⇔ ≥0 49 49(3a + 1) ⇒ 12a − ≥ 12b − ≥ 12c − ≥ 12d − ≥ 12 + Trường hợp 49 48 ⇒ + 3d < ⇒ > 12 48 + 3d 49 Xét tương tự với biến lại ta tìm điều phải chứng minh Đẳng thức xảy a = b = c = d = d< Bài toán 14 [Vasile Cirtoaje, Algebraic Inequalities – Old and New Method] Cho a, b, c số thực dương thỏa mãn a + b + c ≥ Chứng minh a5 − a b5 − b c5 − c + + ≥0 a + b + c b5 + a + c c + b + a Chứng minh Bất đẳng thức tương đương với 1 + + ≤ 2 2 a +b +c b +a +c c +b +a a + b2 + c2 Từ suy ta cần chứng minh trường hợp a + b + c = đủ Áp dụng bất đẳng thức AM-GM ta có 2a 2a ≤ = a5 2 a +1 a Đặt a = x, b = y, c = z lúc ta có x + y + z = ta phải chứng minh 1 1≥ + + 3 2x 2y 2z − x+3 − y+3 − z+3 x +1 y +1 z +1 x +1 y +1 z +1 ⇔1≥ + + 2 2x − x + 2x + y − y + y + 2z − z + 2z + x +1  3− x  ⇔ ∑ − ÷≥ 2x − x + 2x +  cyc   ( x − 1) (−2 x + x +  ⇔ ∑ ÷≥ cyc  6(2 x − x + x + 3)  Không tính tổng quát giả sử x ≥ y ≥ z ⇒ x ≥ ≥ z Xét hai trường hợp + Trường hợp y + z ≥ ⇒ x ≤ ta có −2 x + x + > 0, −2 y + y + > 0, −2 z + z + > Dẫn đến toán hiển nhiên đúng + Trường hợp y + z ≤ ⇒ x ≥ ta có 10 Phần Một dạng biểu diễn thú vị Ở chúng muốn nói đến dạng biểu diễn theo tổng Đây tư tưởng đơn giản giúp ta tìm nhiều lời giải ấn tượng Bây ta chú ý đến đẳng thức sau ak + bk + ck ak bk ck 1= k = + + a + bk + c k a k + bk + c k a k + bk + c k a k + bk + c k Đẳng thức tưởng chừng điều hiển nhiên, không mang nhiều ý nghĩa lại có vai trò quan trọng việc chứng minh lớp bất đẳng thức mà chúng nêu Ở phần kỹ thuật xác định hệ số không thực trước xuất lũy thừa p Nếu sử dụng những biến đổi thông thường phức tạp Vì công cụ mà chúng ta chọn đạo hàm Trước hết xin nhắc lại định lí sau Định lí Fermat Giả sử hàm số f ( x) xác định [a, b] có cực trị địa phương x0 ∈ [a, b] Khi f có đạo hàm x0 f / ( x0 ) = Định lí Roll Giả sử f :[ a, b] → ¡ liên tục khả vi (a, b) Nếu f ( a) = f (b) / tồn x0 ∈ (a, b) cho f ( x0 ) = Bài toán 21 [Võ Quốc Cẩn] Tìm số k > tốt để bất đẳng thức sau đúng với số a, b, c số thực dương a b c + + ≥ 2 2 2 k +4 ka + (b + c) kb + (c + a ) kc + (a + b) Chứng minh Cho a = 1, b = c = ta có k ≤ Ta chứng minh giá trị k tốt để bất đẳng thức đúng Bất đẳng thức cần chứng minh a b c + + ≥1 2 2 a + 2(b + c ) b + 2(c + a ) kc + (a + b) Ta phải xác định hệ số k cho bất đẳng thức sau đúng a ak ≥ k k k a + 2(b + c ) a + b + c Ở ta chuẩn hóa b = c = để việc việc xác định hệ số đơn giản Khi ta cần xác định hệ số k cho a ak ≥ k ⇔ a k + − 2a k + a ≥ a +8 a +2 Đặt f ( a) = a k + − 2a k + a Lại có f (a ) ≥ 0, f (1) = nên theo định lí Fermat ta có f / (1) = Tiến hành đạo hàm f ( a) suy f / (a ) = (k + 2)a k +1 − 4ka k −1 + 2a Theo thi ta có f / (1) = (k + 2) − 4k + = ⇔ k = Như ta dự đoán bất đẳng thức sau đúng 19 a ≥ a4 a + 2(b + c ) a + b + c Sau hoàn thành xong bước dự đoán chúng ta có nhiều đường để lựa chọn Thông thường phép biến đổi tương đương mang lại hiệu bất đẳng thức phụ đúng Nên nhớ bất đẳng thức phụ dự đoán mà thôi, không đúng ngược lại Từng toán ta “tùy ứng biến” Tất nhiên nhiều toán không áp dụng theo cách Chúng ta tiếp tục quay lại toán với phép chứng minh cho bất đẳng thức phụ Theo bất đẳng thức Holder ta có  b + c  từ ta phải chứng minh b + c ≥2  ÷   4 bất đẳng thức a a + 8t ≥ 3 a4 a4 + t ⇔ a + t ≥ a a + 8t ⇔ t ( a − t )2 ≥ b+c Vậy bất đẳng thức hiển nhiên đúng Đẳng thức xảy a = b = c a = t > 0, b = c = hoán vị Nhận xét Quá trình tìm kiếm hệ số k thông qua việc đánh giá theo bất đẳng thức AM-GM sau a ak ≥ k ⇔ a k + − 2a k + a ≥ ⇔ a k + + a ≥ 2a k a +8 a +2 Ở t = Mặt khác theo bất đẳng thức AM-GM a k + + a ≥ a k + Như ta có cần xác định k cho a k + = 2a k ⇔ a k + = a k ⇔ k + = 4k ⇔ k = Bài toán 22 [IMO 2001] Cho a, b, c số thực dương Chứng minh a b c + + ≥1 2 a + 8bc b + 8ca c + 8ab Chứng minh Bằng cách làm tương tự, ta thiết lập bất đẳng thức sau a a4/3 ≥ /3 / 4/ a + 8bc a + b + c Thật vậy, sử dụng bất đẳng thức AM-GM, ta có b / + c / ≥ 2b / 3c / = 2t / , ta cần chứng minh a / + 2t / ≥ a1/3 a + 8t ⇔ 4t / (a / − t 2/ ) ≥ (ñuùng) Do đó, bất đẳng thức đúng Sử dụng tương tự cho b, c cộng lại, ta có đpcm Đẳng thức xảy a = b = c = b → 0, c → 20 Bài toán 23 Cho a, b, c số thực không âm Chứng minh a3 b3 c3 + + ≥1 a + (b + c )3 b + (c + a ) c + ( a + b)3 Chứng minh Tương tự ta có xác định bất đẳng thức phụ sau: a3 a2 ≥ (*) a + (b + c )3 a + b + c Có thể chứng minh bất đẳng thức phụ theo nhiều cách: Cách (*) ⇔ 2a (b + c ) + (b + c ) ≥ a(b + c)3 Điều hiển nhiên đúng, (b + c) a (b + c ) 2a (b + c ) + (b + c ) ≥ a (b + c) + ≥2 = a (b + c)3 4 Cách Theo bất đẳng thức AM-GM ta có (1 + k ) + (1 − k + k ) k2 + k = (1 + k )(1 − k + k ) ≤ = 1+ 2 Áp dụng bất đẳng thức phụ ta có a3 1 a2 ≥ ≥ ≥ = 2 2 a + (b + c )3 a + b2 + c2  b + c  1+ b + c b+c + 1+   ÷ ÷ a2 2 a   a  Áp dụng tương tự với biến lại Cộng vế theo vế ta có có điều phải chứng minh Đẳng thức xảy biến có biến dần Bài toán 24 Cho a, b, c số thực dương Chứng minh a3 b3 c3 + + ≥ 3 3 3 a + (b + c) b + (c + a) c + (a + b) Chứng minh Sử dụng bất đẳng thức Cauchy-Schwarz, ta có 1 a3 VT ≥  ∑ 3  cyc a + (b + c )3  ÷ ≥ ÷  Bất đẳng thức chứng minh Đẳng thức xảy a = b = c Phần Giải một số toán mà điều kiện liên quan mật thiết đến Đa phần toán xét đến có điều kiện mà biến liên hệ với ko k k k k chặt Thường điều kiện dạng a1 + a2 + + an −1 + an = n Tức ta tách theo biến để tìm bất đẳng thức phụ Tuy nhiên với số toán mà điều kiện thiết lập k  n  mối quan hệ “bền chặt”  ∏ ÷ việc tìm bất đẳng thức phụ tương đối  i =1  khó khăn ta đánh giá theo biến nữa Và để áp dụng U.C.T những toán chúng ta phải dùng đến số tính chất hàm số 21 Bài toán 25 Cho a, b, c số thực dương thỏa mãn abc = Chứng minh a b+c b c+a c a+b + + ≥ b + c +1 c + a +1 a + b +1 Chứng minh Áp dụng bất đẳng thức Holder ta có  a b + c b c + a c a + b   a (b + c + 1)  + +  ÷ ∑ ÷ ≥ (a + b + c) ÷ b+c   b + c + c + a + a + b +   cyc Do ta cần phải chứng minh a (b + c + 1) ( a + b + c ) ≥ 2∑ b+c cyc a b a ⇔ ∑ a + 3∑ +3∑ +6 ≥ 4∑ ab + 4∑ a + 2∑ cyc cyc b cyc a cyc cyc cyc b + c Áp dụng bất đẳng thức AM-GM ta có a b a a b ≥∑ ab, ∑ ≥∑ ab, 2∑ ≤ ∑ + ∑ ∑ cyc b cyc a cyc b cyc cyc a cyc cyc b + c Từ ta có a b VT − VP ≥ ∑ a + ∑ + ∑ − 4∑ ab − 4∑ a + cyc b cyc a cyc cyc cyc   ≥ ∑ a + ∑ ab − 4∑ a + = ∑  a − 4a + + ÷ a  cyc cyc cyc cyc  Xét hàm số f ( x ) = x − x + + + ln x với x > ta có x 1  f / ( x) = ( x − 1)  x + + − ÷ x x  1 Nếu x < ≥ , x ≥ ⇒ > f / ( x) = ⇔ x = x x x Từ đễ dàng kiểm tra f ( x) ≥ f (1) = 0, ∀x > Hay x − x + + ≥ −2 ln x, ∀x > x Như ta có   ∑  a − 4a + + ÷ ≥ −2∑ ln a = a  cyc  cyc Bài toán giải Đẳng thức xảy a = b = c = Bài toán 26 [Lê Hữu Điền Khuê, THPT Quốc Học, Thành phố Huế] Cho a, b, c số thực dương thỏa mãn abc = Chứng minh 1 + + ≥1 2 3a + (a − 1) 3b + (b − 1) 3c + (c − 1) Chứng minh Xét hai trường hợp sau 22 + Trường hợp Nếu ba số a, b, c tồn số không lớn Giả sử ⇒ 3a + (a − 1) ≤ Khi bất đẳng thức hiển nhiên đúng + Trường hợp Cả ba số a, b, c không nhỏ ta xét hàm số sau Giống phần trước ta có thiết lập bất đẳng thức phụ dạng 1 ≥ + k ln x 2 x + ( x − 1) Ở ta có qui hàm số mũ chú ý ln x + ln y + ln z = Tiếp tục quan sát thấy đẳng thức xảy a = b = c = Từ ta có phải xác định k cho f / (1) = f ( x) = + ln x − x + ( x − 1) 3 Với x > Khi ta có 2(16 x − 16 x − x + 1) 2( x − 1)(16 x − 1) / f ( x) = = x(4 x − x + 1) x(4 x − x + 1) / Từ suy f ( x) = ⇔ x = 1, x > Dễ dàng kiểm tra f ( x) ≥ f (1) = 0, ∀x > Điều tương đương với 1 ≥ − ln x, ∀x > 2 x + ( x − 1) 3 Sử dụng bất đẳng thức phụ theo biến a, b, c cộng vế theo vế ta có 1 + + ≥ − ∑ ln a = 2 2 3a + (a − 1) 3b + (b − 1) 3c + (c − 1) cyc Bất đẳng thức chứng minh Đẳng thức xảy a = b = c = , a → ∞, b → ∞, c → 0+ hoán vị Nhận xét Bài toán lời giải ấn tượng Vasile Cirtoaje Xin trình bày lại lời giải Sử dụng bất đẳng thức phụ sau 1 2a (a − 1) ≥ ⇔ ≥0 3a + (a − 1) 2a + (4a − 2a + 1)(2a + 1) Điều hiển nhiên đúng với số thực không âm Tương tự với biến lại suy điều phải chứng minh số a Ta có a ≤ Bài toán 27 [Gabriel Dospinescu] Cho a1 , a2 , , an số thực dương thỏa mãn a1a2 an = Chứng minh a12 + + a22 + + + an2 + ≤ 2(a1 + a2 + + an ) Chứng minh Xét hàm số sau với x >   f ( x) = x + − x +  − ÷ln x 2  Khi ta có 23 ( x − 1)  −2 x + x − − x 2( x + 1)    ⇒ f / ( x) = ⇔ x = f / ( x) = 2 x 2( x + 1)( x + x + 1) Qua f / ( x ) đổi dấu từ dương sang âm nên f / ( x) ≤ f (1) = 0, ∀x > Điều có nghĩa   x2 + ≤ x −  − ÷ln x, ∀x > 2  Sử dụng bất đẳng thức phụ cho n biến cộng vế theo vế ta có   a12 + + a22 + + + an2 + ≤ 2(a1 + a2 + + an ) −  − ÷(ln a1 + ln a2 + + ln an ) 2  n   = 2(a1 + a2 + + an ) −  − ln ∏ ÷  i =1  = 2(a1 + a2 + + an ) Vậy bất đẳng thức chứng minh Đẳng thức xảy a1 = = an = Nhận xét Bài toán giải bất đẳng thức phụ quen thuộc x + ≤ 2( x − x + 1) ⇔ ≤ ( x − 1) , ∀x > Sử dụng bất đẳng thức cho n biến cộng lại ta có n   a12 + + a22 + + + an2 + ≤ 2(a1 + a2 + + an ) +  n − ∑ ÷ i =1   ≤ 2(a1 + a2 + + an ) Bất đẳng thức giải hoàn toàn Bài toán 28 [Algebraic Inequalities – Old and New Method] Cho a, b, c số thực dương thỏa mãn abc = Chứng minh a + b + c + 9(ab + bc + ca ) ≥ 10(a + b + c) Chứng minh Ta có cần xác định hệ số k cho bất đẳng thức sau đúng a + 9bc = a + ≥ 10a + k ln a a Tương tự phần trước ta có tìm k = −17 Ta có chứng minh f ( a) = a + − 10a + 17 ln a ≥ a Thật 17 2a − 10a + 17 a − (a − 1)(2a − 8a + 9) f / (a ) = 2a − − 10 + = = a a a2 a2 f / (a) = ⇔ a = Từ đây, ta dễ dàng thấy f (a ) ≥ f (1) = 0, ∀a > hay a + − 10a ≥ −17 ln a a Sử dụng tương tự với b, c cộng lại vế theo vế, ta có đpcm Đẳng thức xảy a = b = c = 24 Phần U.C.T mở rộng Ngay từ đầu viết ta xét đến việc xác định hệ số m theo cách h(ai ) ≥ f (ai ) + ma k + n k k k Với điều kiện xác định toán a1 + a2 + + an = n Tuy nhiên với cách xác định đối với số toán lại không mang lại hiểu Điều hoàn toàn không tốt Vì thúc chúng ta tìm dạng xác định hệ số khác Một cách trực quan chúng ta phân tích toán cụ thể để thấy những nêu Bài toán 29 [Tạp chí Crux, Canada] Cho a, b, c số thực dương thỏa mãn a + b + c = Chứng minh 1 + + ≤ − ab − bc − ac Chắc hẳn từ đầu vào chứng minh toán bạn nghĩ đến việc thiết lập bất đẳng thức phụ dạng 8 ≤ + mx + n ⇒ ≤ + m( x − 1) 9− x 9− x Dễ dàng dự đoán m = Nhưng đáng tiếc với m bất đẳng thức hoàn toàn không đúng kể tư tưởng chia trường hợp phần áp dụng Thật 7+ x ( x − 1) ≤ ⇔0≤− 9− x 8(9 − x) Tuy nhiên U.C.T có tác dụng trường hợp ý tưởng mẻ Hãy chú ý đến cách thiết lập bất đẳng thức phụ sau ≤ + m( x − 1) + n( x − 1) (*) 9− x Việc xác định hệ số bất đẳng thức đòi hỏi chặt chẽ lập luận nới lỏng miền nghiệm biến khiến cho toán không đúng Có nhiều hệ số thỏa mãn để tạo thành đại lượng bình phương ( x − 1) ta phải xác định cho dấu bất đẳng thức đúng Ta có   (*) ⇔ ≤ ( x − 1)  m( x + 1) + n − ÷ (**) 9− x  Từ phân tích rõ ràng ta phải xác định n theo m cho xuất nghiệm x = để hình thành đại lượng ( x − 1) , tức 1 m( x + 1) + n − =0⇔n= − m( x + 1) ⇒ n = − 2m 9− x 9− x Từ vào (**) ta có 1   (**) ⇔ ≤ ( x − 1)  m( x + 1) − 2m + − ÷ 9− x  ⇔ ≤ ( x − 1) (72m − 8mx − 1) Dễ thấy việc xác định hệ số không đơn giản trước Nó đòi hỏi ta phải tìm những ước lượng chặt chẽ để bất đẳng thức không đổi chiều Ta chú ý đến điều kiện toán để tìm ước lượng “tốt nhất” Chú ý > max{ab, bc, ca} ≥ 25 ≥ max{ab, bc, ca} ≥ Tuy nhiên đối với toán cần sử dụng điều kiện yếu mà Đầu tiên đưa số nhận xét sau: Đầu tiên ta cần xác định hệ số m để bất đẳng thức đúng với ∀x ∈ [0,3) Ta thấy trường hợp m < nhận bất đẳng thức ngược chiều cho x = , tất nhiên điều ta mà không mong muốn Vậy dự đoán m ≥ , 72m − − 8mx ≥ 72m − − 24 = 48m − 1 1 ⇒n= Ta cần có 48m ≥ ⇔ m ≥ Vậy nên ta dự đoán m = 48 48 12 Công việc dự đoán hoàn tất Bây ta thử chứng minh xem có đúng thật không Và ta có bất đẳng thức phụ sau x + x + 43 ( x − 1) (3 − x) ≤ ⇔0≤ 9− x 48 48(9 − x) Điều hiển nhiên đúng Áp dụng bất đẳng thức phụ với biến ab, bc, ca ta có 1 1 43 + + ≤ (a 2b + b c + c a + 4ab + 4bc + 4ca ) + − ab − bc − ac 48 16 Ta cần phải chứng minh bất đẳng thức sau a 2b + b c + c a + 4ab + 4bc + 4ca ≤ 15 Đặt k = ab + bc + ca , áp dụng bất đẳng thức AM-GM bất đẳng thức Schur ta có  4x -  k ≤ 3, abc ≥ max 0,  Ta xét hai trường hợp sau   + Trường hợp Nếu x ≤ a 2b + b 2c + c a + 4ab + 4bc + 4ca = (ab + bc + ca ) + 4(ab + bc + ca ) − 6abc 81 225 = k + 4k − 6abc ≤ + = < 15 16 16 + Trường hợp Nếu x ≥ a 2b + b c + c a + 4ab + 4bc + 4ca = (ab + bc + ca ) + 4(ab + bc + ca ) − 6abc nhiên chưa phải đánh giá “tốt nhất” ta làm chặt nữa = k + 4k − 6abc ≤ k + 4k − 2(4k − 9) = ( k − 1)(k − 3) + 15 ≤ 15 Bất đẳng thức chứng minh Đẳng thức xảy a = b = c = Qua trình nhận xét phân tích hi vọng bạn hiểu cách tìm hệ số Ở toán sau không thật cần thiết, việc thiết lập bất đẳng thức phụ đưa cách khái quát Chúng ta đến với toán sau Bài toán 30 [Moldova TST 2005] Cho a, b, c số thực dương thỏa mãn a +b + c = Chứng minh 1 + + ≤1 − ab − bc − ca Chứng minh Với ≤ x ≤ , ta có x + x + 12 ( x − 1) (3 − x) ≤ ⇔0≤ 4− x 15 15(4 − x) 26 3 < nên ta có 2 3 2(a 2b + b c + c a ) + ab + bc + ca + 36 + + ≤ − ab − bc − ca 15 Áp dụng bất đẳng thức AM-GM Cauchy-Schawrz ta có a 2b + b c + c a ≤ a + b + c = Lại có max{ab, bc, ca} ≤ ab + bc + ca ≤ 3( a 2b + b 2c + c a ) ≤ Cộng các bất đẳng thức phụ vế theo vế ta có điều phải chứng minh Đẳng thức xảy a = b = c = Nhận xét Đây toán không khó có nhiều cách tiếp cận khác 1 Áp dụng bất đẳng thức quen thuộc sau + ≥ bất đẳng thức AM-GM x y x+ y Ta có 1 1  1  + ≥  ≥ = , ∀a, b ∈ [0, 2]  2 ÷ 2 ÷ 2 4−a − b   − a − b  − 2ab − ab Qui toán chứng minh 1 + + ≤1 2 4−a 4−b − c2 Sử dụng bất đẳng thức phụ sau a + 15 ≤ − a2 18 Ngoài ta có cách trực quan dễ thực qui đồng sử dụng bất đẳng thức Schur Bài toán 31 [Phạm Kim Hùng] Cho a, b, c, d số thực dương thỏa mãn a +b + c + d = Chứng minh 1 1 + + + ≤2 − abc − bcd − cda − dab Chứng minh Đây toán khó việc thiết lập hệ số phải cần những đánh giá chặt chẽ suy luận hợp lí Chúng ta phân tích đường đến lời giải toán Ta xác định hệ số m, n cho ≤ + m( x − 1) + n( x − 1), ∀ ≥x≥0 3− x 3 Như phân tích ta tìm n = − 2m , bất đẳng thức cần chứng minh tương đương với ( x − 1) (6m − − 2mx) ≥ Dễ dàng kết luận m ≥ 16 6m − − 2mx ≥ 6m − − m 3 Ta cần có 27 16 m≥0⇒m≥ 16 3 6− 3 5 m≥ = 16 14 Do > nên ta cần có 6− 5 Từ ta chọn m = ⇒ n = − từ ta có bất đẳng thức phụ sau 14 14 x − x + 12 ( x − 1) (8 − x) ≤ ⇔0≤ 3− x 14 14(3 − x) 8 > ≥ x≥0 Điều hiển nhiên đúng với 3 6m − − Sử dụng bất đẳng thức phụ chú ý max{abc, bcd , cda, dab} ≤ 3 suy ta cần chứng minh 5( a 2b c + b c d + c d a + d a 2b ) − 3(abc + bcd + cda + dab ) ≤ Có thể chứng minh bất đẳng thức nhiều cách Sau xin trình bày cách dựa vào kỹ thuật hàm lồi a2 + b2 c +d Đặt t = ,k = , x = ab, y = cd đó, ta có t ≥ x, k ≥ y Bất đẳng thức 2 cần chứng minh tương đương với f ( x) = 10 x k + 10 y 2t − x y + 2k − y x + 2t − Ta có f // ( x ) = 20k + 3y (2 x + 2t )3 ≥0 Suy f ( x ) hàm lồi, f ( x) ≤ max{ f (t ), f (0)} Ta có ( )( ) f (0) = yt + yt − ≤ yt ≤ 3 < f (t ) = 10 y 2t − yt + 10k 2t − 3t y + 2k − = g ( y ) Tương tự ta có g ( y ) hàm lồi nên g ( y ) ≤ max{g (k ), g (0)} Ta có 8 < g (0) = kt + 5kt − ≤ kt ≤ 3 k + t2 g (t ) = 4(kt − 1)(5kt + 1) ≤ kt ≤ =1 Vậy bất đẳng thức chứng minh xong ( )( ) Ngay từ ban đầu chúng nói bất đẳng thức không dễ đòi hỏi những đánh giá chặt chẽ U.C.T đóng vai trò bàn đạp quan trọng để đến lời giải 28 Bài toán 32 [Võ Quốc Cẩn] Cho số thực a, b, c, d thỏa a + b2 + c + d = , chứng minh bất đẳng thức 1 1 16 + + + ≤ − ab − bc − cd − da Chứng minh Tương tự toán trước, ta thiết lập bất đẳng thức sau với x≤ ≤ 32 x + 10 1− x Từ đây, ta suy 1 1 32 40 + + + ≤ ( a 2b + b c + c d + d a ) + − ab − bc − cd − da 9 32 40 = (a + c )(b + d ) + 9 40 16 ≤ (a + b + c + d ) + = 9 Từ đây, ta có đpcm Đẳng thức xảy a = b = c = d = ± Nhận xét Bài toán đặt để “làm mạnh” toán sau Phạm Văn Thuận 1 1 1 + + + + + ≤8 − ab − bc − cd − da − ac − bd với giả thiết Lời giải tác giả cho toán dài phức tạp, dùng U.C.T mở rộng ta lại có lời giải ngắn gọn đơn giản! Ngoài ra, chúng ta có cách “làm mạnh” khác cho toán Phạm Văn Thuận, ta có 1 1 1 + + + + + ≤8 2 2 2  a+b b+c c+d  d +a  a+c b+d  1−  ÷ 1−  ÷ 1−  ÷ 1−  ÷ 1−  ÷ 1−  ÷             Bài toán bạn ZhaoBin, sinh viên người Trung Quốc đưa lời giải đẹp cách sử dụng bất đẳng thức Cauchy-Schwarz, đây, chúng xin giới thiệu lời giải khác theo tư tưởng U.C.T Sử dụng bất đẳng thức bài, ta cần chứng minh (a + b) + (a + c) + (a + d ) + (b + c) + (b + d ) + (c + d ) ≤ Thật vậy, ta có (a + b) = 2(a + b + 6a 2b ) − (a − b) ≤ 2(a + b + 6a 2b ) Tương tự với số hạng lại, ta suy VT ≤ 6(a + b + c + d ) = Bất đẳng thức chứng minh xong Thật tự nhiên, câu hỏi sau đặt ra, liệu bất đẳng thức sau có đúng? 1 1 16 + + + ≤ 2 2  a+b b+c c+d  d +a 1−  ÷ 1−  ÷ 1−  ÷ 1−  ÷         Thật đáng tiếc bất đẳng thức lại không đúng! Các bạn cần cho a = b = 0.4, c = d = 0.84 29 Bài toán 33 [Vasile Cirtoaje] Cho số không âm a, b, c, d có tổng 4, chứng minh bất đẳng thức sau 1 1 + + + ≤1 − abc − bcd − cda − dab Chứng minh Ta thiết lập bất đẳng thức sau với ≥ x > 48 ≤ x + x + 10 5− x Do +, Nếu max{abc, bcd , cda, dab} ≤ sử dụng bất đẳng thức trên, ta cần chứng minh a 2b 2c + b c d + c d a + d a 2b + abc + bcd + cda + dab ≤ Bất đẳng thức dễ dàng chứng minh dồn biến dùng hàm lồi +, Nếu max{abc, bcd , cda, dab} ≥ , không tính tổng quát, giả sử abc ≥ , đó, chú ý với x, y ≥ 0, x + y ≤ 5, ta có 1 1 xy (10 − x − y ) + − − = ≥0 5 − x − y − x − y 5(5 − x)(5 − y)(5 − x − y ) Suy 1 1 + ≤ + 5− x 5− y 5− x− y Và đó, với x, y , z ≥ 0, x + y + z ≤ ta có 1 1 + + ≤ + 5− x 5− y 5− z 5− x− y − z 2 Chú ý ∑ (a b c + abc) ≤ abc ≥ nên bcd + cda + dab < , cyc 1 1 + + ≤ + − bcd − cda − dab − d ( ab + bc + ca ) Ta cần chứng minh 1 + ≤ − abc − d (ab + bc + ca ) Đặt x = − d , abc ≥ nên ≥ x = a + b + c ≥ 3 abc ≥ 3 , theo bất đẳng thức AMGM abc ≤ x , ab + bc + ca ≤ x 27 Do đó, ta cần chứng minh 1 + ≤ 5− x − x (4 − x) 27 Hay f ( x) = x − x − 80 x3 + 360 x − 675 ≤ Ta có f / ( x ) = x ( x − 4) + x ( x − 12) + 48 x(15 − x) < Suy f ( x ) hàm nghịch biến, f ( x) ≤ f (3 2) = 27(48 − 77) < Từ đây, ta có đpcm Đẳng thức xảy a = b = c = d = Phần Lời kết 30 Sau trình tìm hiểu phân tích cụ thể toán, hẳn bạn phần cảm nhận nét đẹp U.C.T thực kĩ thuật đơn giản dễ hiểu Chúng không xem U.C.T phương pháp thống mà đơn giản kĩ thuật cần biết cần nắm vững bạn học bất đẳng thức Nhiều người quan niệm U.C.T ý nghĩa theo thân chúng nên khái quát để sử dụng số trường hợp U.C.T bước đệm quan trọng mang nhiều ý nghĩa đường tìm lời giải cho toán Một kĩ thuật hay không thiết nó giải tất dạng toán mà phải đưa ta đến những ý tưởng, đường sáng sủa, dễ nghĩ, dễ nhận thấy mặt trực quan Trong chuyên đề nhiều toán hình thức cồng kềnh USAMO 2003, JMO 1997, những toán không khó, không chọn đúng hướng làm dẫn đến những lời giải chấp nhận đúng mặt toán học Đó những toán đại diện cho U.C.T kết hợp với kĩ thuật chuẩn hóa Tuy nhiên chưa phải điểm dừng Ở phần tiếp theo, xuất nhiều toán mang đậm sắc tức sử dụng mỗi U.C.T không đích Cách khắc phục phân chia trường hợp để giải Đây sở để tìm khoảng nghiệm cần xét biến Việc đánh giá đòi hỏi người làm tinh tế khéo léo phần trước Tuy nhiên bạn có niềm tin chuyện giải Sau nắm tay những kiến thức định kỹ thuật chúng ta bước qua khoảng không gian phức tạp dùng U.C.T để giải những toán mà điểm cực trị đạt chỗ Bao gồm trường hợp tất biến trường hợp có (n − 1) biến khác biến lại Ở ta chú ý đến bất đẳng thức Vornicu Schur để khắc phục nhược điểm U.C.T Phần kĩ thuật phân tách theo tổng dạng đẹp kỹ thuật này, số toán tiêu biểu cho dạng phân tách IMO 2001 số toán nêu Dù U.C.T dùng theo tư tưởng khác với phần trước Như bạn biết U.C.T thông thường biết đến với toán mà biến số độc lập không liên quan đến Tuy nhiên xét với lớp toán chưa lột tả hết nét đẹp kĩ thuật đơn giản Ta sử dụng U.C.T để tìm những bất đẳng thức phụ với điều kiện liên quan mật thiết với Tức không tách theo đơn lượng biến để giải U.C.T đòi hỏi bạn phải có những kiến thức hàm số để tìm ước lượng chinh xác Cuối chúng ta đến số toán khó mà theo nhiều người quan niệm giải U.C.T, điều điểm yếu kỹ thuật Khi việc thiết lập hệ số thắt chặt chuyện khác Như bạn thấy U.C.T mở rộng mang những đặc điểm phức tạp hiệu mang lại bất ngờ Một số toán khó đưa dạng đơn giản để giải theo số phương pháp biết Đó nét độc đáo kỹ thuật Tuy nhiên hẳn chưa phải U.C.T “chặt” Còn nhiều điều nữa kỹ thuật chờ bạn khám phá Chúng xin kết thúc viết Hi vọng với những dòng tâm bạn bất đẳng thức phần gợi mở cho bạn giúp bạn tìm thêm những ý tưởng sáng tạo mới, những hiểu biết Và quan niệm đằng sau lời giải cho mỗi toán trình dự đoán, thử, sai đúng Hẹn gặp lại bạn ngày không xa Phần 10 Bài tập áp dụng 31 Bài toán [Diễn đàn toán học] Cho a, b, c, d , e số thực không âm thỏa mãn a + b3 + c3 + d + e3 = Tìm giá trị nhỏ biểu thức a2 b2 c2 d2 e2 + + + + − a − b3 − c − d − e3 Bài toán [Vasile Cirtoaje, Crux Mathematicorum, Problem 3032] Cho a, b, c số thực không âm thỏa mãn a +b + c = Chứng minh 1 + + ≤ − ab − bc − ca Bài toán [Mathematical Excalibur, Vol 9, Num 1, 8/2004] Cho a, b, c, d số thực dương thỏa mãn a + b + c + d = Chứng minh 6(a +b +c + d ) ≥ (a +b +c + d ) + Bài toán [Mihai Piticari, Dan Popescu, Old and New Inequalities] Cho a, b, c số thực dương nhỏ thỏa mãn a + b + c = Chứng minh 6(a +b +c ) + ≥ 5(a +b +c ) Bài toán [Titu Andreescu, Gabriel Dospinescu, Old and New Inequalities] Cho a, b, c số thực dương nhỏ thỏa mãn a + b + c = Chứng minh 1 27 + + ≤ 2 1+ a 1+ b 1+ c 10 Bài toán [Andrian Zahariuc, Old and New Inequalities] Cho a, b, c ∈ (1, 2) Chứng minh b a c b a c + + ≥1 4b c − c a 4c a − a b 4a b − b c Bài toán [Vũ Đình Quý] Cho a, b, c số thực dương thỏa mãn abc = Chứng minh 1 + + ≤3 a − a +1 b − b +1 c − c +1 Bài toán [Vasile Cirtoaje] Cho a, b, c, d số thực dương thỏa mãn abcd = Chứng minh 1+ a 1+ b 1+ c 1+ d + + + ≤4 + a2 + b2 + c2 + d Bài toán [Vasile Cirtoaje, GM-B,11,1999] Cho a, b, c, d số thực dương thỏa mãn abcd = Chứng minh 1 1 + + + ≥1 3 1+ a + a + a 1+ b + b + b 1+ c + c + c 1+ d + d + d 32 Bài toán 10 [China TST 2004] Cho a, b, c, d số thực dương thỏa mãn abcd = Chứng minh 1 1 + + + ≥1 2 (1 + a ) (1 + b) (1 + c) (1 + d ) Bài toán 11 [Arkady Alt, Crux mathematicorum] Chứng minh với a, b, c > ta có a b c a / + b2 / + c2 / + + ≥ 3 a+b b+c c+a Complete • • Võ Quốc Cẩn Nguyễn Thúc Vũ Hoàng Trong viết có sử dụng nhiều toán trích dẫn từ • Algebraic Inequalites – Old and New Method tác giả Vasile Cirtoaje • Sáng tạo Bất đẳng thức tác giả Phạm Kim Hùng • Old and New Inequalities tác giả Titu Andreescu, Vasile Cirtoaje, G Dospinescu, M Lascu 33 ... a ) 4(3 − a) Điều hiển nhiên đúng a ∈ [0,3) Sử dụng bất đẳng thức cho b, c cộng lại, ta có đpcm Bài toán 10 [Phạm Văn Thuận, Mathlinks forum] Cho a, b, c số thực dương Chứng minh (b + c −... x − 1) ( x + x − 1) ≥ (ñuùng) Từ đây, suy −1 +, Nếu c ≥ , sử dụng bất đẳng thức trên, ta có đpcm 13 −1 , có khả xảy ++, Nếu b ≤ , ta có +, Nếu c ≤ 3 3  a − 3a + =  a − ÷ + ≥ 2 4  b − 3b... 1099t − 546t + 117) ≥0 (t − 3t + 3)2 (4t − 6t + 3) Dễ dàng kiểm tra bất đẳng thức đúng, ta có đpcm Đẳng thức xảy a = b = c = f // ( x) = Bài toán 17 [Mở rộng từ Poland 1996] Cho a, b, c số thực

Ngày đăng: 21/04/2017, 09:38

Từ khóa liên quan

Tài liệu cùng người dùng

Tài liệu liên quan